Practice Questions from Supplement

Pataasin ang iyong marka sa homework at exams ngayon gamit ang Quizwiz!

Attorney Mort represents Julio, who was injured in an auto accident caused by Yvonne. Julio has back pain and headaches. When Mort interviewed Julio and asked him how frequently he had headaches, Julio said, "I don't have them every day, and sometimes they aren't bad. But about 4 days a week, I wake up with a severe headache." Mort told Julio that minimizing the frequency of his head pain would weaken his case. Julio asked Mort what he should say. Mort said, "When I ask you about your headaches, you can say: 'Most mornings, I wake up with severe headaches." Julio said this in the deposition, and he was not asked more questions. The case was settled. Julio got into a dispute with Mort about fee calculations, and Julio filed a bar complaint. During the investigation, Julio described how Mort had coached him to testify. Is Mort subject to discipline for his conduct regarding Julio's testimony?

- no, b/c he did not tell Julio to lie -Rule 3.3(a)(3): bars a lawyer from counseling a client to testify falsely, but it does not bar a lawyer from coaching a client to tell the truth in a way that is most favorable to the client

Valladia, Inc., is a small, closely held corp. Valladia and its president, Alan, are defendants in a civil action brought by a state attorney general who accuses both of them of fraud. Both of them desire to be represented by Martha, an attorney. Martha will bill each defendant for services provided. Martha reasonably believes that she could provide competent and diligent representation to both. Martha advises the BoD of Valladia of all the foreseeable risks of the corporation being represented by the same attorney who is representing Alan, and she advises Alan of all of the foreseeable risks of being represented by the lawyer who is representing the board. Valladias BoD votes to approve the representation, and Alan approves as well. Martha writes a letter to the corporation and to Alan noting that both have given their informed consent to her representation of both parties. May Martha represent both of them?

- yes, b/c both defendants gave informed consent to the joint representation -Rule 1.7(a)(2): concurrent conflict -Rule 1.7(b)(1) -There is a significant risk that the representation of Alan will be materially limited by Martha's representation of the corporation, b/c the corporation is paying her fee. Therefore, a concurrent conflict exists. -Martha reasonably believes that she can provide competent and diligent representation to both and both affected clients have given informed consent, confirmed in writing -All of the requirements of Rule 1.7(b) are met, and the representation may be undertaken

Attorney Kristin is a state prosecutor, and she is prosecuting Oliver Burson for the murder of his ex-wife. Before the trial begins, a reporter asks to interview Kristin for a television report. Which of the following statements would Kristin be well-advised NOT to make during the interview?

-"Oliver Burson, who murdered his ex-wife, will be prosecuted to the fullest extent of the law" -Rule 3.6 cmt 5: prosecutors should not comment on the defendants guilt or innocence because such a statement is more likely than not to have a material, prejudicial effect -Rule 3.6(b)(1): a lawyer may publicly state the claim or defense involved and the identity of the persons involved -Rule 3.6(b)(7): provides that in a criminal case, a lawyer may publicly state the identity, residence, occupation, and family status of the accused and the fact time and place of the arrest

Jaiden, an assistant general counsel of Plenum, Inc., a pharmaceutical manufacturer, was asked to investigate if the research division of Plenum was concealing reports of adverse reactions to the company's best-selling product, Somalox. Two users had contacted the general counsel's office, reporting that they had submitted adverse reaction reports to the research division, as directed on the package insert, but that they had received no response. Jaiden schedules a meeting with Bertha, the chief of the research division. Jaiden asks her to show him adverse reaction reports that the company has received. Bertha flushes and stammers in response; "I am afraid that I have let a few of these slip through the cracks. Since you are the company's lawyer, I can speak to you in confidence, right? Am I in trouble here?" In answering Berthas questions, which of the following statements would Jaiden be best advised to make?

-"i represent Plenum, so I cannot give you any legal advice except the advice to get a lawyer" -Rule 1.13, cmt 10: times when the organizations interests may be or become adverse to those of one or more of its constituents -Rule 4.3

Pablo is a "notario," a non-lawyer who helps Spanish-speaking low-income undocumented immigrants with their immigration problems. He charges much less for legal advice than a lawyer would charge. One day, Pablo telephones Ann, an immigration lawyer. Ann and Pablo are friendly because both of them serve truly indigent clients for no charge at all. Pablo says, "I've got a guy in my office, from Honduras, who wants to apply for asylum, but he has two misdemeanor convictions for possession of drug paraphernalia, for which he was sentenced, in each case, to a month in jail. He wants to know whether that is an absolute bar to getting asylum, in which case he would decide not to apply. What's the answer?" Ann has all the clients she wants at present and does not want to suggest that the man become her client. Which of the following responses is best?

-"i would like to help you, but I can't, because it is improper for me to assist a notario in providing legal advice" -Rule 5.5(a): lawyers may not assist another in practicing law in violation of the regulation of the legal profession in the jurisdiction in question -With very limited exceptions, all jurisdictions prohibit non0lawyers from giving legal advice, so Ann should not assist Pablo in giving legal advice no matter how well-intentioned either Pablo or Ann is

Attorney Sebastian graduates from law school and starts his own criminal defense practice. He establishes an LLC for his practice and sets up a bank account for the office expenses. He also has a personal bank account. He decides to charge the relatively low fee of $200 per hour for his work. Within the first month of opening his practice, five different clients retain him in connection with their criminal matters. Pursuant to the retainer agreements, each client gives Sebastian a check for $4,000. Sebastian will earn $200 for each hour that he works. After 20 hours, he will bill each client for additional sums. At a minimum, how many additional bank accounts must Sebastian open?

-1 -Rule 1.15 -Sebastian must keep his own funds separate from the property of clients. Until he has done the work, the $4.000 he receives from each client is still the property of the client. Accordingly, Sebastian needs one additional account: a trust account for client property

Attorney Leticia represents client Benjamin, a pharmacist, in an employment discrimination suit. With the help of his best friend Rocky, who happens to be sleeping on his couch that month, Benjamin writes a summary of the case, which Benjamin then emails to Leticia. Which one of the following statements is correct?

-Benjamin or Rocky can be compelled to testify about the communication -the communication is only protected by the attorney-client privilege if the communication was for the purpose of giving or receiving legal advice or legal services (which it was) and the client reasonably believed the communication was confidential -Here Benjamin asked his pal Rocky to help him draft the statement. Accordingly, he could not have believed the statement was confidential. The participation of a third person would not result in the loss of privilege if the third person was necessary to facilitate the communications -There is no indication that Benjamin needed help. He just happened to get it. Since the privilege would be considered to be waived by the participation of a third person, either Benjamin or Rocky could be compelled to testify about the contents

Eduardo is a sole practitioner. He wants to handle the following matters and to make agreements with clients under which he would receive 20 percent of any recovery awarded to the client. In which of these cases would Eduardo be subject to discipline if he undertook the representation under those terms?

-Celeste has asked Eduardo to file an action seeking an order requiring her ex boyfriend, who is the father of her daughter, to pay child support -Rule 1.5(d): a lawyer shall not enter into an arrangement for, charge, or collect: (1) any fee in a domestic relations matter, the payment or amount of which is contingent upon the securing of a divorce or upon the amount of alimony or support, or property settlement in lieu therefor A contingent fee is not permitted in this case b/c the action is a petition for child support. The support order has not yet been issued, so the amount has not yet been determined. The rule seeks to ensure that the amount of awards in domestic relations matters will not be inflated by the lawyers desire for larger fees

Geraldine, an attorney in a legal aid program, provides pro bono representation to Amber, who is indigent, disabled, and homeless, in litigation against Mike's Job Counseling Service. Amber had paid $300 to Mikes, which did not give her any job leads or help. Mikes has recently been exposed in the local paper for not actually having helped anyone to get a job. Winter is approaching, and it has become increasingly difficult for Amber to live on the streets. She is in danger of freezing to death. All of the homeless shelters in the area are full. Geraldine cares about Amber and wants to keep her from freezing. Which of the following statements is correct?

-Geraldine may give Amber $500 so that she can rent a modest room -Rule 1.8(e): prohibits lawyers from providing financial assistance to clients in connection with pending or contemplated litigation except for court costs and expenses of litigation -Geraldine is suing Mike's Counseling Service on behalf of Amber. Any financial support of Amber during this time would be considered to be "in connection with litigation" -Rule 1.8(e)(3): allows a lawyer who is representing a client pro bono to 'provide modest gifts to the client for food, rent, transportation, medicine, or other basic living expenses'

During his final year of college, Henry got excellent grades and scored high on the LSAT> However, he had 2 car accidents, ran up $11,500 in credit card debt and was fired from his job after failing to show up for 10 days. Henry was evaluated and diagnosed with bipolar disorder. Henry's doctor explained that he went through a manic period his last year of college. Doc prescribed meds, and ever since Henry's illness is considered well-managed. He is told it will not affect his ability to practice law. Henry started law school 3 months after finishing college. Henry is applying for admission to the bar. Henry believes his disorder does not affect his ability to practice law in a competent, ethical, and professional manner, and his physician agrees and has supplied him with a statement. Which of the following is most accurate?

-Henry must disclose the diagnosis and provide detailed information about it -applicants must answer bar application questions fully and truthfully -Question 1 of the character and fitness questionnaire asks for information about conduct w/in the last 5 years that would call into question an applicants fitness to practice -Henry's sudden run-up of debt and his absence from work, apparently caused by his disorder, could reflect on his fitness to practice law. The "thorough explanation" of the conduct required by question 1 would require disclosure of his diagnosis

Ian, a lawyer, is trying to build up his client base. He frequently reads in his local newspaper about automobile accidents in which pedestrians are injured. He would like to visit the victims, either at hospitals or their homes, within a few days after the accidents, before insurance adjusters contact those potential plaintiffs and persuade them to sign settlements in which they receive only small amounts of compensation. He would represent these clients on a contingent-fee basis. Ian knows that the ethics code imposes some restrictions on solicitation, but he thinks he has a right to reach out to these victims before the insurance adjusters get to them. Assuming that none of the victims are people known to Ian or his partners, and that none of them are lawyers, may Ian visit the victims as he desires?

-No, b/c Ian is pursuing this work to acquire fee-generating cases -Rule 7.3: prohibits in-person solicitation by lawyers if a significant motive for the solicitation is pecuniary gain. -The rule allows in person solicitation even motived by a desire for income if the person contacted by the lawyer is herself a lawyer, is a person with whom the lawyer or the law firm has a prior close personal or business relationship, or the person is a regular user of legal services of the type offered -Pedestrian accident victims' would not be 'regular users' of such services, so this answer correctly states that this type of contact is forbidden

Attorney Priya represents Janice in her suit against major canned goods company. Priya asks Janice if they can meet the next day to talk about what terms Janice wants. When they meet, Priya tells Janice that she thinks she could get at least $4 million dollars in a settlement. Janice is thrilled, and she tells Priya, "I didn't think we'd get that much! You have been such an amazing lawyer and friend throughout this process. If you think that we can get more by rejecting the offer, I trust your judgment. Do what you think is best." The next day, Priya meets with opposing counsel, who offers $3.25 million to settle the case. W/o conveying the offer to Janice, Priya responds, "My client rejects your client's offer." The case goes to trial, and Janice loses and recovers nothing. She complains to the bar. Is Priya subject to discipline for not telling Janice about the offer?

-No, b/c Janice authorized Priya to reject the settlement offer -Rule 1.2 cmt 3: a client may authorize the lawyer to take specific action on the clients behalf without further consultation. Absent a material charge in the circumstances and subject to to Rule 1.4, a lawyer may rely on such an advance authorization -Rule 1.4 cmt 2: a lawyer who receives from opposing counsel an offer of settlement in a civil controversy must promptly inform the client of its substance unless the client has authorized the lawyer to accept or reject the offer

Spencer desires to represent his golfing partner Craig, who was injured on a ski slope that was apparently negligently maintained. Spencer tells Craig on the phone that he is willing to work on a contingent fee basis, under which Craig will be charged nothing unless Spencer obtains a settlement or wins a judgment. In either case, Spencer would charge 33 percent of any recovery after the deduction of the expenses of litigation. Spencer explains that Craig will not be responsible for any litigation expenses unless there is a recovery that exceeds the amount of the expenses. They discuss these points, and Craig agrees to the fee arrangement. Spencer contacts the ski slope's insurer, and within three weeks, with Craig's approval, the case is settled for $21,000. Spencer transmits $14,000 of that settlement to Craig. Spencer spent only four hours on the case. Was Spencer's conduct proper?

-No, b/c he did not disclose all the details of the fee arrangement in writing signed by Craig -Rule 1.5(c) requires any contingent fee agreement to be in writing signed by the client

Maude, a lawyer, was retained by Anya, who lives in assisted-living. Anya noticed that many residents were contracting respiratory infections and she noticed the ceiling tiles have turned black. She suspected that Warner Management, Inc., who runs the building, was cutting costs, and had not been servicing the air systems often enough. Maude wants to question Xavier, the employee who services the air systems. If Xavier reveals that he failed to regularly service the system or change the filters, Warner could be liable. Maude knows that Warner is represented by attorney Peter in all matters related to the building. Maude is planning to tell Xavier that she represents Anya but is not planning to tell him that he has a right to consult with counsel of his choice. Also, she does not plan to ask Peter's permission to question Xavier, or even to notify Peter. May she question Xavier without getting Peter's permission?

-No, b/c his failure to change the filters often enough could be imputed to the organization for the purpose of civil liability -Rule 4.2: bars a lawyer from communicating about the subject of a representation with a person the lawyer knows to be represented by another lawyer in the matter w/o that lawyers consent -Rule 4.2 cmt 7: explains that in a case of a representative organization such as a corporation, this rule 'prohibits communications w/ a constituent of the organization who supervises, directs or regularly consults with the organization lawyer concerning the matter or has authority to obligate the organization w/ respect to the matter or whose act or omission in connection with the matter may be imputed to the organization for purposes of civil or criminal liability -B/c Xavier's acts or omissions may be imputed to the organization for purposes of liability, Maude needs Peter's permission b/f interviewing him

Eleanor represents Carmichael Inc. The corp. has 5 stockholders; 4 are Carmichael family members, and the 5th is an outside investor. While reviewing corporate documents, Eleanor discovers that Marcus, has been harvesting cell phone numbers from e-mails that went through the company's server and selling them to telemarketing companies. This information, if it became public, would be very damaging to reputation. She consults Carmichael's president, Joseph, who says that he has suspected this might be happening, but that Eleanor should just leave it alone. Eleanor suspects that Joseph may be collaborating with Marcus. Eleanor goes to a meeting of the BoD that is attended only by Joseph and his wife and son, who are also board members, and Joseph again tells her to leave it alone. The company is regulated by the Public Service Commission. Do the rules require Eleanor to reveal the misconduct to the commission?

-No, b/c rule 1.13 does not require such disclosure, and the disclosure would violate rule 1.6 -Rule 1.13(c)(2): when the lawyer for a corporation reasonably believes that a violation is reasonably certain to result in substantial injury to the corporation she represents, and the highest authority that can act for the corporation fails to address the situation, the lawyer 'may' reveal the information to prevent the injury to the corporation, -but still not REQUIRED to do so

Esteban, a lawyer in State A with a general practice, regularly represents and advises Patrick who lives in State B. Patrick owns and operates a supply store in State A. For years, Esteban has advised him about employment and tax issues related to his business. Patrick wants to start a second business and decided to locate this enterprise in State B. He asked Esteban if he would meet with him to advise him on State B tax and real estate law to assist him in launching the new business. Esteban has the knowledge to provide competent legal advice to Patrick on these issues even though he is not a member of the bar of State B. He would not charge Patrick fees that were higher than those permitted by State B. Esteban has not mentioned to Patrick that he is not licensed in State B; it did not cross his mind. Would Esteban be subject to discipline if he provides legal advice to Patrick as Patrick requests?

-No, b/c the advice arises out of Esteban's practice in State A -Rule 5.5(c): a lawyer may provide temporary legal services in a jurisdiction in which he is not admitted if the services 'arise out of or are reasonably related to the lawyer's practice in a jurisdiction in which the lawyer is admitted to practice -Rule 5.5 cmt 6: services may be temporary, even though the lawyer provides services in his jurisdiction on a recurring basis or for an extended period of time -Esteban's advice is temporary b/c Esteban is providing it only to help Patrick start up his business rather than on an ongoing basis. It is related to his practice in State A b/c he already provides business advice to Patrick in State A

Samir practices family law. His clients often have problems that might better be addressed by a marital counselor. He wants to form a partnership with his cousin Veena, a licensed social worker. They would share a suite of offices, and a client could receive services from either or both of them, as the client preferred. Samir and Veena want to share the fees earned by either of them, in proportion to the work done by each. Samir will take steps to ensure that Veena complies with the Rules of Professional Conduct. Veena will not direct or regulate Samir's professional judgment in rendering legal services, and all clients will be informed that only Samir is licensed to practice law. However, Samir does not plan to seek each client's written consent to receive legal services from a partnership that includes a social worker who is not a lawyer. May they create this partnership?

-No, b/c the partnership would be prohibited in any event -Rule 5.4: prevents a lawyer from forming a partnership with a non-lawyer, regardless of how the fees are divided or what disclosures are made to clients -Rule 5.4(b): a lawyer shall not form a partnership with a nonlawyer if any of the activities of the partnership consist of the practice of law

Margery owns and manages a law practice. Her specialty is employment discrimination cases alleging gender discrimination. Recently, she has gotten a substantial number of referrals. At the end of every calendar year, Margery directs her office manager to send a $50 bottle of champagne to every individual who referred a new client to her practice that year to thank them for sending clients her way. The recipients include other lawyers, clients, former clients, and friends, relatives, or any others who have referred matters to her. Giles, an estate planning lawyer, referred a prospective client to Margery whom she represented in an administrative matter. Giles, who does not drink alcohol, received a bottle of champagne from Margery. Giles files a complaint against Margery with the local bar counsel for giving gifts to get business. Is Margery subject to discipline?

-No, because a $50 bottle of champagne is not greater in value than a token item that might be given for the holidays or for other ordinary social hospitality -Rule 7.2: a lawyer shall not compensate, give or promise anything of value to a person for recommending the lawyer's services -Rule 7.2 CMT 4: paragraph (b)(5) permits lawyers to give nominal gifts as an expression of appreciation to a person for recommending the lawyer's services or referring prospective client. The gift may not be more than a token item as might be given for holidays or other ordinary social hospitality

Five years ago, attorney Barry worked at a large law firm with offices across the country. One of the firm's partners defended Panko, Inc., an appliance manufacturer, against allegations that its toasters had a defect that would cause many of them to catch fire. Barry was not involved in that litigation and did not learn anything about it during his time at the firm. Barry has since opened up a private practice of his own. Ted comes to Barry's new office seeking legal help. Ted tells Barry that his Panko toaster recently overheated and caused a house fire. He would like to sue Panko for damages. May Barry represent him in a suit against Panko w/o Panko's informed consent?

-Yes, b/c Barry did not actually acquire confidential information that is material to Ted's lawsuit while he was working at his old law firm -Rule 1.9(b): a lawyer shall not knowingly represent a person in the same or a substantially related matter in which a firm with which the lawyer was formerly associated had previously represented the client (1) whose interests are materially adverse to that person and (2) about whom the lawyer had acquired information protected by Rule's 1.6 and 1.9(c) that is material to the matter -Barry moved from one law firm to another. His former firm represented Panko in a similar matter

Leta graduated from law school 1year ago and is applying for admission to the bar. While in law school, Leta accumulated $150,000 of debt from law school loans and credit card expenses related to her wedding. Upon graduation, Leta turned down a high-paying job at a prestigious law firm, choosing instead to work for $12 per hour at a public interest organization that represents indigent people. She plans a career in consumer credit and bankruptcy law. She hopes that her work at the organization will eventually lead to a full-time public service job. Accordingly, she has not been able to begin paying off her debt. One credit card company has obtained a judgment against her for $7.000. The state bar has denied her application for admission to the bar because it finds that her conduct with respect to her finances indicates that she is not fit to practice law. Is the state bar's position legally sustainable?

-Yes, b/c a court could conclude that her conduct with respect to her personal finances indicates that she is not fit to practice law -Each state conducts a character and fitness evaluation as part of the bar application process, and the states have broad discretion to deny admission. There have been instances in which individuals were denied bar admission based on failure to pay loans. These decisions are arguably misguided or unfair, but they are within the states discretion

Assume that the lawyers primary motive is to earn money. Which is prohibited by the Model Rules?

-a lawyer is walking on the beach during a summer evening. He sees a drunk man throw his girlfriend against the wall, causing her to lose consciousness briefly. The boyfriend leaves the scene. When the woman comes to, the lawyer gives her his card and offers to represent her in a suit against the man on a contingent fee basis ` -Rule 7.3(a): prohibits a lawyer from soliciting professional employment by live person-to-person contact if a significant motive for doing so is pecuniary gain with an exception if the prospective client has a family, close personal, or prior business or professional relationship with the lawyer or law firm or where the prospective client routinely uses for business purposes the type of legal services offered by the lawyer

An example of a statement that is protected by the attorney-client privilege if an adverse party sought to compel the lawyer to disclose that information

-a paralegal interviews a client about his case, and she gives her notes to her supervising attorney -the paralegal is interviewing the client as an agent of the attorney, and therefore the conversation is privileged

Example of a communications that is protected by the attorney-client privilege

-a prospective client tells an attorney the facts of her case, but the attorney decides not to take the case -this conversation is protected by the attorney-client privilege, b/c conversations b/w prospective clients and lawyers are privileged

The state in which Stella practices recently enacted an animal cruelty law. Stella represents Chix, Inc., a poultry company, in a suit. The suit alleges that the company has violated the new state law and seeks punitive damages. Stella files a motion urging that the new law does not allow recovery of punitive damages. In an unrelated matter, Stella represents a group of students, seeking to recover punitive damages against the school for allowing dissection of live animals in violation of the statute. When Stella agreed to represent the students, she explained to them that she was arguing against the availability of punitive damages in the Chix case. Stella is representing the students pro bono. Which of the following factors is LEAST relevant to determining the risk that Stellas actions on behalf of one client would materially limit Stellas effectiveness in representing the other?

-c the fact that Stella would not benefit financially if the statute were interpreted to allow punitive damages -Rule 1.7(a)(2) conflict of interest -Rule 1.7 cmt 24: the mere fact that advocating a legal position on behalf of one client might create precedent adverse to the interests of a client represented by the lawyer in an unrelated matter does not create a conflict of interest. Factors relevant in determining whether the client needs to be advised of the risk include: where the cases are pending, whether the issue is substantive or procedural, the temporal relationship b/w the matters, the significance of the issue to the intermediate and long term interests of the clients involved, and the clients reasonable expectations in retaining the lawyer

Attorney Aria represents Jaya, who is a defendant in a securities fraud prosecution. One day, Aria is interviewing Jaya about the allegations that she engaged in insider trading. Jaya, who is married, brings a man named Josh to the interview, and she reveals to Aria that she is having an affair with him. She further reveals that Josh provided her with some inside information. Josh is not a defendant in the case and is not represented by Aria. Jaya's statements about Josh are:

-confidential, but not privileged -Rule 1.6: prohibits lawyers from revealing information relating to the representation of a client -Thus, this information is confidential -Privilege only covers communications the client reasonably believes were made in private b/w client and lawyer for the purpose of seeking or delivering legal advice or legal services

Attorney Moira represents defendant Oleg in a murder trial. The police discovered the victim's body in a ditch and found Olegs fingerprints on a kitchen knife that was under the body. Before the trial commences, Oleg tells Moira that he did not murder the victim, but he knows that his son did, because his son confessed to him. He tells her that under no circumstances will he turn his son in, and he does not want his son to know that he told Moira of his son's confession. He refuses to testify at his trial. What is Moiras best course of action?

-do not reveal the information and do not tell Oleg's son that she is aware of his confession to his father -No exception to the rule against revealing confidences applies

A law firm has offices in 3 major U.S. cities. Once a year, lawyers from all 3 offices convene for a firm retreat. One of the partners, Sandler, represents Nigel, a celebrity chef, in several lawsuits. Sandler prepares a complex counseling exercise for a retreat, based on his relationship with Nigel, called "dealing with a difficult client." Sandler does not change Nigel's name or any of the facts in any of the training materials. The materials provided detailed descriptions of the cases in which Nigel was a party and included stories about difficult interactions b/w the chef and his lawyers. The materials are distributed in loose-leaf notebooks clearly marked 'confidential training materials.' Nigel learns about the training program from Tommy, one of the firm's junior lawyers, whom Nigel is doing casually (unbeknownst to Sandler). Nigel is furious. Is Sandler subject to discipline for violation of rule 1.6?

-no -Rule 1.6 cmt 5: lawyers in a firm may, in the course of the firms practice, disclose to each other information relating to a client of the firm, unless the client has instructed that particular information be confined to specific lawyers. -Sandler's preparation of materials using client information w/o changing the client's name, for wide dissemination w/in the firm, may have been unwise. However, b/c the rules allow sharing of confidence w/in a firm, irrespective of the size of the firm, this conduct would not be a basis for discipline. -Sandler's notation on the cover of the materials, stating that they were confidential, took account of Rule 1.6(c) which requires reasonable efforts to prevent improper disclosure of confidences

Attorney Lilith represents Opal in a custody case against Opal's ex-husband, Claude. Lilith knows that Claude is represented by another lawyer, Oscar. Claude enters Lilith's office one day and tells her that he wants to speak with her about Opal's treatment of their children. Lilith says, "As you know, Oscar represents you in this matter." Claude says, "I know that, but I want to talk to you anyway." May Lilith allow Claude to continue?

-no, b/c she does not have consent from Claude's lawyer -Rule 4.2: bars communications b/w lawyers and persons known to be represented with respect to a matter, w/o consent of that persons lawyer

Eleanor represents Carmichael Inc. The corp. has 5 stockholders; 4 are Carmichael family members, and the 5th is an outside investor. While reviewing corporate documents, Eleanor discovers that Marcus, has been harvesting cell phone numbers from e-mails that went through the company's server and selling them to telemarketing companies. This information, if it became public, would be very damaging to reputation. She consults Carmichael's president, Joseph, who says that he has suspected this might be happening, but that Eleanor should just leave it alone. Eleanor suspects that Joseph may be collaborating with Marcus. Eleanor goes to a meeting of the BoD that is attended only by Joseph and his wife and son, who are also board members, and Joseph again tells her to leave it alone. The company is regulated by the Public Service Commission. Must Eleanor withdraw from representation of Carmichael?

-no b/c nothing in rule 1.16 requires withdrawal -Rule 1.16: a lawyer may withdraw for various reasons, including if: (a) a client persists in a course of action, involving the lawyers services that the lawyer reasonably believes is criminal or fraudulent, (b) the client has used the lawyers service's to perpetrate a crime or fraud, or (c) the client insists upon taking action the lawyer considers repugnant or with which the lawyer has a fundamental disagreement. the lawyer is NOT REQUIRED to withdraw for any of these reasons

On a dark night in a bad part of town, a young man named Alan is found dead. Chad is arrested and put in jail, accused of murdering Alan. Chad tells Alberto, his lawyer, that he could not have murdered Alan, because at the time of the murder, he was burying the body of another man, Ethan, whom he had killed. Ethan has been declared missing, and the police are investigating whether foul play has occurred. Chad tells Alberto where he buried Ethan. Obviously, Chad does not want anyone to know his alibi because then he would be charged with another murder. In exchange for the alibi information, Alberto could obtain a desirable plea bargain for Chad on the current and prospective charges. May Alberto reveal this information to the prosecutor without consulting Chad?

-no, b/c Alberto obtained this information in the course of representing Chad -Rule 1.6 -no exception to the confidentiality rule of 1.6 applies

Salima represents Anthony, a man accused of stealing money from his company. Anthony was a bookkeeper. His duties included administering the payroll system. Anthony altered a computer program so he would get paid extra. Anthony's theft was noticed during an audit. Anthony tells Salima that he also rigged the program so that he would also receive money that should have been paid to Jason. He says that after he was caught, Anthony re-programmed the system so that the checks would be properly paid going forward. Neither Jason nor the company are aware. Salima meets with the company's general counsel to discuss restitution, who says that the audit is complete and presents an accounting that lists only the money that Anthony stole from the company. Salima reasonably believes that unless she reveals the theft, there will be no restitution to Jason. May Salima reveal this information without Anthony's consent?

-no, b/c Anthony did not use Salima's services to steal Jason's money -Rule 1.6(3): only applies when the client used the lawyer's services to commit the crime or fraud

Armand, a lawyer, represented Walter, a plastic surgeon, during Walter's contested divorce from his wife five years ago. Armand no longer has any contact with Walter, and he does not remember anything about Walter's case. Recently, one of Walter's patients, Celia, developed serious complications and nearly died as the result of the extensive cosmetic surgery that Walter performed on her. She wants to sue him for punitive damages based on gross negligence. She has asked Armand to represent her. May he do so without Walter's consent?

-no, b/c Armand would normally have obtained information in the divorce case that could be helpful to Celia -Rule 1.9: a former lawyers client may not represent a person in a substantially related matter in which the new client's interests are adverse to the former clients interests, unless the former client consents -Rule 1.9 cmt 3: a matter is substantially related if there is a substantial risk that confidential factual information as would normally have been obtained in the prior representation would materially advance the new clients position in a subsequent matter -Armand would normally have learned a great deal about the extent and location of Walter's assets during the divorce proceedings. It would benefit Celia to have ready access to that information, especially b/c in a punitive damages case, the recovery could well exceed the amount of Walter's malpractice insurance

Attorney Betsy has more clients than she can handle and wants to expand her law firm. She would like to hire two associates but does not have enough capital to do so. Banks will not loan her enough money to pay the associates for the year or so until their fees would cover their own costs. But Betsy's father Roberto, who is a successful investment banker and not a lawyer, would like to provide Betsy with one million dollars to help her expand her business. In return, Roberto would become a limited partner of Betsy, entitled to receive two percent of all of the gross profits of the firm for as long as it exists. Betsy would like to enter into this agreement with Roberto. She and Roberto agree that Roberto will never try to influence Betsy's judgment in rendering legal services. May Betsy enter into this agreement?

-no, b/c Betsy may not enter into this arrangement even though Roberto would never try to influence Betsy's professional judgment -Rule 5.4: prohibits lawyers from allowing non-lawyers to invest in their firms -Rule 5.4(d): a lawyer shall not practice with or in the form of a professional corporation or association authorized to practice law for a profit if: (1) a nonlawyer owns any interest therein

Jill, an attorney, brings a lawsuit on behalf of her client, Ann, against Grant, after Grant fails to repay Ann's loan to him of $5,000. The retainer provides that Jill's contingent fee will be 30 percent of any recovery. Jill files the complaint one day too late, and the suit is dismissed with prejudice. Jill is chagrined and embarrassed and wants to make amends for her mistake. She would like to pay Ann out of her own pocket without disclosing her error to Ann or to anyone else. She would simply get Ann a cashier's check for the portion of the $5,000 that would have been paid to her if the court had ordered Grant to repay the $5000. May she do so if she avoids making any false statement about the source of the funds?

-no, b/c Jill is required to tell Ann that she missed the deadline and that the case was dismissed -Rule 1.4 -a lawyer must keep a client informed about the status of a matter. The fact that the client's case was dismissed is a significant event in the litigation. A lawyer who makes a serious mistake must inform the client (and should inform her malpractice insurer) of her mistake

Last year, attorney Mira represented Sally, the owner of a small pizza restaurant, in a suit against Rome's Own, Inc., a supplier of shredded cheese, based on breach of contract. The parties settled the claim for $15,000. Mira and Sally remained friendly after the matter was settled, and have gone to the movies and to dinner a few times in the last year. Earlier today, Sally's next-door neighbor, Jose, came into Mira's office for a consultation and asked Mira to represent him in a civil suit against Sally. Jose alleges that Sally's pet macaw bit him, causing an infection and permanent scarring. Jose wants to get a court order against Sally requiring that the bird to be euthanized pursuant to a state law. Sally already paid for his medical care. Jose does not want further damages. Must Mira obtain Sally's informed consent before she can agree to represent?

-no, b/c Jose's lawsuit is not substantially related to the breach of contract lawsuit against Rome's Own, Inc. -Rule 1.9(a): an attorney must obtain a former client's informed consent ONLY IF she plans to represent another person 'in the same or substantially related matter in which that person's interests are materially adverse to the interest of the former client -Since Jose is only seeking an injunction, it is unlikely that any prior confidences could be used adversely. Therefore, there probably is not a substantial relationship between the prior matter and the current one, so no consent is required

Attorney Meghan represents Walter, a used-car salesman who has been criminally charged with driving while intoxicated. Before Walter's trial, a friend of Meghan's, Laurel, asks Meghan to represent her in her breach of contract suit against Walter and his company. If Meghan were to sue Walter on behalf of Laurel, another lawyer would defend Walter in that matter. Meghan has formed a close bond with Walter as a result of her work on his criminal case, and she reasonably believes that she could not represent Laurel very vigorously, because of her friendship with Walter. She explains her concern to Laurel, and Laurel states that she understands the risks and agrees to the representation anyway. She confirms her understanding in writing. Walter also gives informed consent in writing. May Meghan represent Laurel?

-no, b/c Meghan does not reasonably believe that she can competently and diligently represent Walter and Laurel, even though both clients gave informed consent, confirmed in writing -Laurel would be representing one client in one suit against another client whom she represents in a different matter. A lawyer may sue a current client on behalf of another client ONLY IF the lawyer reasonably believes that she can represent both clients competently and diligently, and if both clients give informed consent, confirmed in writing. -Meghan does not reasonably believe she can competently and diligently sue Walter on behalf of Laurel while simultaneously representing him on a criminal charge

Spencer, a businessman, calls Kai, an attorney. He says he is looking for a lawyer so that he can sue his former business partner, Max, with whom he has had a falling out. He asserts that he has proof that Max was stealing money from his company and covering it up by keeping two sets of books. Spencer says he has photos of several pages from both sets of books, which he obtained by sneaking into Max's house, where he found the books in the study and photographed them with his cell phone. Kai makes an appointment to meet with Spencer the following week to discuss possible representation. The day after the phone call, Max calls Kai to ask for representation because he thinks Spencer is going to sue him. Max offers Kai a generous flat fee for the work. Kai would like to represent Max. May Kai represent Max without Spencer's consent?

-no, b/c Spencer has given Kai information that could be used adversely to Spencer if Kai represents Max -Rule 1.18(c) -Spencer is a prospective client. Kai may not represent Max b/c he has received information from Spencer about his unlawful break-in. That information could be used adversely by Spencer if Spencer sues Max.

Attorney Uma is a solo practitioner who represents Royce in a civil case. She is scheduled to argue an important motion in three days. She learns that her father's funeral will take place on the same day as the hearing. The judge denied her motion for a continuance, because she has previously been granted rote postponements of the argument of this motion. Uma's good friend Wendell is a seasoned litigator. She reasonably believes that he can competently represent Royce at the hearing, so she asks him if he can handle the hearing for her, and he agrees. He agrees to treat the file as confidential and to return it to Uma promptly after the hearing. Uma leaves a phone message for Royce, but he doesnt return her call. She gives Wendell Royces file so that he can familiarize himself with the case. Assume that Wendell can competently represent the client at the hearing. Was it proper for Uma to give Wendell the file?

-no, b/c Uma did not obtain Royce's prior consent -Rule 1.6(a) prohibits a lawyer from revealing information relating to the representation of a client w/o client's consent. -Rule 1.6 cmt 5: explains that a lawyer practicing in a law firm is impliedly authorized to disclose client confidences to other lawyers w/in the law firm, but not to lawyers outside the firm

Attorney Yoshi represented client Cyrus in his divorce. During the course of the representation, Cyrus told Yoshi that if his ex-wife, Tisha, was awarded their beachfront vacation home in Florida, he would burn it down when it is vacant, because he could not stand the thought of his ex-wife using the vacation home without him. Yoshi advises Cyrus that this is a terrible idea. He points out that deliberate arson could lead to criminal charges and civil liability, not to mention termination of visitation rights. He also cautions that Cyrus could injure or kill a member of his family if he burns down that house. Cyrus repeats to Yoshi that he would not do this if anyone was in the house, but he does not assure Yoshi that he won't torch the building. The divorce was finalized, and Tisha was awarded the vacation home. Yoshi has terminated his lawyer-client relationship with Cyrus. Yoshi reasonably believes that revelation

-no, b/c Yoshi learned this information during the course of the representation -Rule 1.6 cmt 20 -the duty to protect confidences learned during the course of the representation continues beyond the termination of the representation, and no exception to the duty to protect confidences applies here

Burke is a trusts and estates lawyer. Ida, an acquaintance, asks him to represent her in a negligence lawsuit against a major retail company. she stands to make millions from the lawsuit, and she is happy to hire Burke on a contingent fee basis. Burke has never before handled a negligence action and he has never handled a trial of any kind. Burke accepts the case. He does not tell Ida that he has no experience in handling cases of this sort whom he could consult, but unfortunately he does not have such contact. He studies the relevant law and procedure. He performs well but loses the case. Is Burke subject to discipline?

-no, because he studied the relevant law and procedure and performed well -Rule 1.1, cmt. 2 -Burke may acquire the knowledge he needs to represent a client in an unfamiliar area through necessary study.

Attorney Tito is a sole practitioner. He represents Laura, a criminal defendant charged with stealing a diamond necklace from Matteo. Matteo and Laura went on a few dates. Laura claims that Matteo gave her the necklace on their second date. On the day of the jury trial, in the elevator on the way to the courtroom, Tito overhears Matteo telling his friend that he gave Laura the necklace, and that the only way he could get it back was to claim that she stole it. Tito has been preparing for the trial for weeks. Tito plans to continue to represent Laura and testify in the trial as to what he heard. He has a co-counsel who could take over the trial while Tito is testifying, but Tito is much better prepared and more experienced. The transfer of primary responsibility for the trial to another lawyer would work substantial hardship on Laura. May Tito continue to represent Laura and testify in the trial?

-no, b/c a lawyer may not serve in the same trial as both advocate and witness -Rule 3.7: bars a lawyer from being an advocate in a trial at which he is likely to be a necessary witness, but an exception permits a lawyer to testify if his disqualification would work substantial hardship on the client

Larry, the father of Linda, Abby, and Carl died. His property was to be divided equally among his children. Most of his property is in cash and stocks, but Larry also left his children a lakeside cabin. The three adult children decide that Carl should take the cabin and should pay his sisters 1/3 of its value. They hire Morton, who reasonably believes that he can competently represent all 3 siblings. He explains the potential problems, advantages, and risks and obtains their consent. While preparing transfer documents, Morton discovers that Carl can obtain a reduction in tax liability if postponed 1year. Linda and Abby are eager to conclude and they will earn more interest if the payment is made this year. Carl asks Morton whether he could slow things down and postpone the transfer of the cabin, w/o revealing any reasons for the delay. May Morton delay the transaction w/o disclosing the reasons to Carl's sisters?

-no, b/c each client is entitled to be told information relating to the representation that might affect the clients interests -Rule 1.7 cmt 31: continued common representation will almost certainly be inadequate if one client asks the lawyer not to disclose to the other client information relevant to the common representation -Morton owes an equal duty of loyalty to each client and each client has the right to be informed of anything bearing on the representation that might affect that clients interests and the right to expect that the lawyer will use that information to that clients benefit

Attorney Mosi is representing Blake, a doctor accused of malpractice. Blake prescribed Amaronset (AM) to Eloise, a patient suffering from migraine . AM is not regulated by the FDA and is not widely used for migraines. Blake didn't tell Eloise that it was experimental. 3 years after her treatment ended, Eloise died from a brain tumor. Her family is suing Blake. A study recently published in Germany found that 1 in 20 of the people who took AM for more than two years developed malignant tumors. During his representation, Blake tells Mosi that he prescribed AM to 2 others. These 2 patients do not know about the recent research or that Eloise died. Blake refuses to inform them. Mosi reasonably believes that it is not reasonably certain that Blakes 2 other patients are at risk of death or substantial bodily harm. Does Mosi have the discretion to reveal this information to Blake's other patients?

-no, b/c even if the two patients might have tumors, Mosi must keep this information confidential -Rule 1.6(b)(1) =the possibility of harm to the patients does not rise to the level required by 1.6(b)(1) to allow revelation

Attorney Suzette is a solo practitioner in a small town. She specializes in medical malpractice cases and has handled hundreds of such cases over the past two decades. Suzette's friend Gunter hosts a local radio program, and he offers Suzette a 15-second advertising spot in his program. Suzette's ad reads, in its entirety: "Have you or has someone you know been hurt by a doctor's negligence? Attorney Suzette Bellows can help. Bellows specializes in medical malpractice cases. Call (555) DOC-HELP now for a free consultation." Gunter plays a recording of the ad on the radio. Is Suzette subject to discipline?

-no, b/c everything in the ad is truthful -Rule 7.1 -the advertisement does not include any false or misleading information, so there is no violation

Lawyer Shawn has handled various property and business transactions for Evelyn, her adult daughter Adelaide, and her business partner, Oliver, during the last decade. 5 years ago, Shawn wrote a will for Evelyn in which she left all of her property to Adelaide. Shawn represented and gave advice to both Evelyn and Adelaide. Adelaide, lives with her mother, suffers from disabling epilepsy and has not been able to support herself. After Shawn finished the will, he drafted a letter to Evelyn and Adelaide informing them that his office was closing their file and confirming the termination of their lawyer-client relationship. However, he failed to mail them the letter. Last week, Evelyn came to Shawn wanting Shawn to write a new will for her in which she would leave all of her property to Oliver. She further asks Shawn not to reveal to Adelaide the contents of the new will. May Shawn write the new will for Evelyn?

-no, b/c he did not obtain Adelaide's informed consent to his representation of Evelyn with respect to the new will -rule 1.9(a) -Shawn may not represent Evelyn in drafting her new will. Adelaide is a former client of Shawn's. The new matter is the same matter as the one on which he previously represented Adelaide, and Evelyn's interests are materially adverse to Adelaide's, so his revising the will is prohibited unless Adelaide gives informed consent -To obtain Adelaide's consent, Shawn needs to reveal confidential information to hear about her mother's intentions as to the will, and he can not do this unless Evelyn consents (which she didn't)

Sophia retained Drake to sue the city after she stepped into a deep hole and broke her leg. She incurred $4,000 in medical bills. State law provides that a city is not liable for personal injury unless the suit is filed within 1year. Drake never filed. 6 months after the 1-year period was up, Sophia sued Drake for malpractice, based on his failure to file. Drake called Sophia 5 days before the one-year period was up and told her that he was ready to file, and Sophia told him not to, saying " I am a finalist for the city's commissioner, and I won't get the job if I sue the city. strictest confidence." Sophia didn't get the job. Drake asserted, in his answer the malpractice complaint, that Sophia told him not to file her suit because she was hoping to get a job as commissioner. He limited the disclosure to the minimum necessary to defend himself. Is Drake subject to discipline?

-no, b/c he revealed no more than necessary to defend himself -Rule 1.6(b)(5): allows lawyers to reveal confidential information to respond to allegations in a lawsuit by a client concerning the lawyers representation of the client -Rule 1.6 cmt 16: provides that a lawyer should make efforts to limit disclosures to the parties and the tribunal

Athena practices law. She lived next door to Meredith, an elderly widow. Meredith lived with her daughter Dora who was dating Sven. Athena didn't do any legal work for Meredith, but knew her well. At Meredith's request, Athena acted as one of two witnesses to Meredith's most recent will. This new will replaced an earlier will which left everything to Dora. The new will left $500,000 to Dora, $1,000,000 to Sven, and the rest to charity. Meredith named Ernest as her executor. Meredith died last week. Dora plans to contest the will. Athena probably will be required to testify. Ernest has asked Athena to represent him and defend the will in the probate proceeding. Athena has never before represented Dora, Sven, or Ernest. Athena wants to represent Ernest in probating the will, and also to testify truthfully that she knew Meredith well and that Meredith appeared to her to be competent. Is her proposed conduct proper?

-no, b/c she is a necessary witness on a contested issue -Rule 3.7: allows a lawyer to serve both as an advocate and a witness where the subject matter of the lawyers testimony is uncontested -this rule bars Athena from acting as an advocate in the proceeding and also testifying as a witness b/c the issue on which she would testify is contested. No other exception applies b/c the matter does not involve legal fees, and the advocate/witness rule imposes no substantial hardship imposed on the client, since Ernest is not yet a client and can retain a different lawyer

Erin, a lawyer, had a motion that was scheduled on Nov 5. Manfred, represented her adversary. On Oct 31, Erin learned that her son's surgery had been scheduled for Nov 5. She tried to call Manfred to discuss rescheduling, but he could not be reached. So without first notifying Manfred, she telephoned the judge's clerk. To her surprise, the judge picked up the phone. Erin explained the situation and asked the judge whether the argument could be scheduled for Nov 12. The judge checked his computer and advised that he could fit in the argument on that date, and he advised Erin to notify Mantred of her request and if Manfred had no objection, to telephone his clerk in a few hours. He said that his clerk would also send a notice to her and to Manfred, describing his conversation with Erin and notifying that in the absence of any objection, the hearing would be rescheduled as requested. Is Erin subject to discipline?

-no, b/c she only discussed a procedural issue with the judge -Rule 3.5: a lawyer shall not communicate ex parte with a judge during the proceeding unless authorized to do so by law or court order -Rule 2.9 Judicial Conduct--describes the exceptional circumstances in which ex parte communications are proper: when circumstances require it, for scheduling purposes, does not address substantive matters, as long as the judge promptly notifies all other parties of the substance of the ex parte communications and gives them opportunity to respond

Attorney Salim represents Roland, a doctor, in a medical malpractice case. In an e-mail, Roland reveals to Salim that on the day in question, he had had a couple of drinks before he performed the surgery at issue. He asks Salim whether that would expose him to punitive damages. Salim replies that it would not under the law of the state. Roland is to testify at the trial. In preparing for his testimony, Roland asks Salim whether, if the opposing counsel asks Roland whether he had anything to drink on the day in question, he can successfully invoke attorney-client privilege to avoid answering the question. Salim's answer should be:

-no, b/c the attorney-client privilege does not shield him from having to answer the question -opposing counsel is questioning Roland about the underlying facts, not about the statement he made to Salim. The privilege protects only communication b/w lawyer and client, not the facts discussed by them

Attorney Charlie works at a law firm of 30 lawyers in a large city. He represents his friend Suma in a wrongful discharge suit against her former employer, an architecture firm, Suma and her husband, Todd, are in the process of divorcing. Suma has retained a family lawyer associated with another firm to represent her in the divorce case. Charlie just learned that one of his law partners, Angus, is representing Todd in the divorce Charlie and Angus agreed not to communicate with one another about either of the matters. They did not speak to Suma and Todd about the conflict. May Charlie continue to represent Suma in her civil suit against her former employer?

-no, b/c the firm did not obtain both clients informed consent, confirmed in writing -Rule 1.10: when lawyers are associated with a firm, none of them may represent a client when one of them practicing alone would be prohibited from doing so by Rule 1.7 or 1.9 -Therefore, one must ask whether one of the these lawyers could reasonably believe that he could competently and diligently represent both clients.

Chris and Bobby are accused of burglary. They want Meyer to represent both of them. They believe if they coordinate their stories and refuse to cooperate with the police, the state won't have enough evidence to convict either of them. Meyer obtains the police reports, charging documents, and both criminal records. Meyer learns that the two are alleged to have had relatively equal roles in the burglary, but Chris may have been the instigator. Chris is 26 years old and Bobby is 19. Chris has 3 prior felony convictions, while Bobby has a prior conviction for possession of a small amount of marijuana, for which he received a suspended sentence that could be revoked if he is again convicted. Meyer's law clerk recommends that he obtain a court order permitting the joint representation, but Meyer declines. May Meyer represent both defendants if each gives informed consent to the joint representation, confirmed in writing?

-no, b/c the joint representation presents a non-consentable conflict -Joint representation of criminal co-defendants always presents a risk that the representation of one client will be materially limited by the lawyer's responsibilities to the other client. Virtually all authorities recommend against such representation -In this instance, either defendant might get a better deal by testifying against the other, producing a genuine conflict. In particular, Bobby could benefit by confirming that Chris was the instigator and testifying or offering to testify against him.

Harriet intends to bring an action in small claims court against her landlord. She meets with lawyer Joaquin, who is experienced in landlord-tenant law, to see if he will represent her in the matter. After discussing the case, Joaquin and Harriet agree to a flat fee of $1000 for the representation, a reasonable fee given the time and effort Joaquin will expend on the case. Joaquin knows that Angela, a landlord-tenant lawyer three blocks away, would do the same work for $500. In fact, Joaquin believes that Harriets case is so strong that she could probably represent herself and win. Joaquin does not inform Harriet of these things, and he accepts the case. Is Joaquin subject to discipline?

-no, because attorneys are not required to tell prospective clients that they could resolve their problem at a lower cost or no cost -The Model Rules do not require a lawyer to reveal to a prospective client that the client might obtain similar services elsewhere for a lower price or the lawyers belief that the client might be able to resolve the problem w/o using legal services at all

Chloe and Matilda are partners in a small law firm. Chloe has represented Speisler Motors for years. She represents Speisler in all litigation in which it is a party. At present she is defending the company in a lawsuit brought by Tom, a driver of a speisler Car who was blinded when an airbag spontaneously exploded. When the bag exploded, Tom's car hit Anton, a pedestrian, who died after the accident. The executor of Anton's estate has asked Matilda to represent the estate in a lawsuit against Tom and Speisler Motors. Matilda and Chloe both believe that they would be able to provide competent and diligent representation to their respective clients. They have agreed not to talk with each other or to share documents. Speisler, Tom, and Anton's executor all give verbal consent to the representation after full disclosure of all material risks. Is it proper for Matilda to accept representation of Anton's estate?

-no, b/c there is a conflict of interest and the conflict is not consentable -Rule 1.7(a) -Rule 1.7(b)(3): this conflict is not consentable -The interests of Anton's estate and Speisler are directly adverse. Representation of the estate would involve the assertion of a claim by Anton's estate (one client) against Speisler (another client represented by the firm in the same litigation)

Attorney Darla represents the plaintiffs in a class action lawsuit against Nutrisnax, a granola bar company, alleging false advertising. The plaintiffs allege that Nutrisnax falsely advertised its granola bars as a healthy alternative to candy bars when, in fact, the granola bars contained just as much fat, sugar, and sodium as the average candy bar. The class includes anyone who purchased a Nutrisnax granola bar in the last three years. Last week, a potential client named Samuel came into Darla's office and asked her to represent him in his claim for child support against his former husband, Franklin. Darla learned that Franklin purchased hundreds of Nutrisnax granola bars during the relevant period, making him one of the unnamed members of the class in the granola bar suit. Must Darla obtain Franklins informed consent before agreeing to represent Samuel?

-no, b/c unnamed members of a class are ordinarily not considered to be clients of the lawyer for conflicts purposes -Rule 1.7 cmt 25: when a lawyer represents or seeks to represent a class of plaintiffs or defendants in a class-action lawsuit, unnamed members of the class are ordinarily not considered to be clients of the lawyer for purposes of applying (a)(1) of this rule. THUS the lawyer does not typically need to get the consent of such a person before representing a client suing the person in an unrelated matter

Attorney Damon represents his friend George in a civil case in which George is a defendant. George is alleged to have driven into the plaintiff's fence negligently. George and Damon meet in Damon's office to discuss the case. After some discussion about their children, who play together, George tells Damon that he was driving under the influence of PCP at the time of the incident. Several months later, George's wife files for divorce, and her lawyer subpoenas Damon to testify. The wife's attorney wants Damon to testify that George told Damon that he drove under the influence of drugs. May Damon be required to testify about George's use of PCP?

-no, b/w the information is privileged -the statement made by George to Damon in the course of seeking legal advice or services, and no exception to the rule against revealing privileged information applies

Barbara approaches attorney Morgan to ask for representation in a landlord/tenant suit. Morgan has not handled this type of case before. She is willing to take it on and will charge a much lower hourly rate than her normal rate, but only if Barbara agrees not to sue her for any mistake she might make. Morgan has accordingly drafted a retainer agreement with Barbara that Barbara waives any potential claims for malpractice against Morgan. Morgan has read Barbara this provision and explained it to her, and she has encouraged her orally to seek the advice of another lawyer about whether to hire Morgan on these terms. Barbara says that she fully understands the provision, that she does not need to consult another lawyer, and that she wants to sign the retainer agreement. May Morgan represent Barbara pursuant to this agreement?

-no, because Barbara was not independently represented in making this agreement -Rule 1.8(h)(1) -a lawyer shall not make an agreement prospectively limiting the lawyer's liability to a client for malpractice unless the client is independently representing in making the agreement. It does not matter that Morgan advised Barbara to consult another lawyer; Barbara must actually be represented by another lawyer in making the agreement

Attorney Dahlia represents restaurant owner Stefan in a suit. During representation. Dahlia learns that Stefan uses PestOFF in his kitchen. PestOFf has recently been shown to cause birth defects if consumed by pregnant women, and is banned in the state. Its use is a misdemeanor. The product was applied only once, and only to the floor of a closet. It was never used near food. There is nothing to suggest any ingestion or injury to a person. A city ordinance requires anyone, including lawyers, who learns of its use to report it to local police. Failure to report is a violation punishable. Dahlia confronts Stefan and insists that he stop using this product and that they report his past use to the police. Stefan stops using but refuses to report, Dahlia then withdraws from representation of Stefan and reports to police. Is Dahlia subject to discipline for reporting Stefans use of PestOP to the police?

-no, because Dahlia's disclosure is permitted under these circumstances -Rule 1.6(b)(60 -Dahlia is permitted to reveal the confidence of this situation b/c revelation is required by law

Johann, a lawyer, is has recently taken Carly's products liability case. They have agreed to a fee of $250/hour. Because he has only handled a few such cases, he wants to avoid being sued for malpractice. He would like Carly to sign a retainer agreement that provides that any malpractice claim that she wants to make against him has to be resolved by an arbitration rather than in a court. He drafts the retainer agreement and explains it to her but does not tell her that she may have a different lawyer advise her about the desirability of agreeing to the arbitration. Arbitration agreements between professional persons and their patients or clients are not prohibited by state law. She signs the agreement. Is Johann subject to discipline?

-no, because Johann explained the effect of the arbitration term to Carly -Rule 1.8 cmt. 14 -the rule "does not prohibit a lawyer from entering into an agreement with a client to arbitrate legal malpractice claims, provided such agreements are enforceable and the client is fully informed of the scope and effect of the agreement. The comment should be understood to mean that an arbitration clause is not a limitation on malpractice liability, and therefore the lawyer need not even advise the client of the desirability of seeking independent legal advice before signing it

Tomas, IWT Corp's lawyer, fired Celinda. IWT claims Celindas written work was sloppy; Celinda claimed IWT simply wanted to give the job to a younger person. She wanted her job back or a cash settlement. Pursuant to the employment contract, it to be resolved through arbitration. While arbitration was pending, IWT's general counsel read the emails Celinda sent Tomas, from the time Celinda received the firing notice until she departed. IWT's handbook says that the e-mail system belongs to the company and that info sent over email may be read or disclosed by company officials. One e-mails said, "I know that my work isn't as good as it used to be, and that I have been making mistakes. So maybe they do have good reason. IWT's general counsel sent a copy of this e-mail to Tomas. Tomas didn't tell Celindas lawyer that he had a copy of the emails, planning to use it during arbitration. Is Tomas subject to discipline?

-no, because no ethics rule requires this disclosure, even if litigation or arbitration is pending -Rule 4.4(b): requires disclosure of electronically stored information relating to the representation of the lawyer client only if the information was inadvertently sent

Client Sylvia hired lawyer Dino to represent her in a personal injury action against Chainmart. In the course of the representation, Sylvia told Dino that she was prepared for the case to go to trial, if necessary, and that she probably would not even consider accepting a settlement offer below $2 million. Dino thought she might be awarded $10 million or more if the claim were decided by a jury. The evening before trial, opposing counsel called Dino and offered a settlement of $1 million. Dino tried calling Sylvia but could not reach her. Five hours after trying unsuccessfully to reach Sylvia and hoping to be able to get some sleep before the trial, Dino called opposing counsel and rejected the offer. The case went to trial the next morning, and after trial, the jury awarded Sylvia $5 million. Is Dino subject to malpractice liability for his actions?

-no, because the jury award was greater than the settlement offer -Rule 1.2 -Sylvia will be unable to demonstrate that she suffered any damages. Dino violated rule 1.2 because he rejected the settlement offer w/o communicating it to his client. However, he would not be subject to malpractice liability for his failure to communicate the settlement offer b/c there are no damages

5 years ago, Tammy was murdered at her home. Her boyfriend Ray was accused. At first, he claimed innocence. After 12 hours of police questioning, he confessed. He retained Ming, a lawyer. Ray was convicted, largely on the basis of his confession, and was sentenced to 40 years in prison. Daniel, a prosecutor for the county, recently negotiated a plea bargain with the attorney for Burt, another man who was alleged to have committed several murders. Burt agreed to plead guilty to manslaughter. Part of the deal is that he would have to provide details of his other crimes. During those discussions, Burt revealed that he had robbed and killed Tammy. He had dated her a year earlier, but she had rejected him in favor of Ray. Burt consents to provide a blood sample, which matches the blood that was found at Tammy's house the day after the murder. Ray's blood has not been found in the house. what, if anything, must Daniel do?

-promptly disclose this evidence to Ming (unless a judge orders a delay in disclosure) and also disclose new evidence to a court or to the chief prosecutor -Rule 3.8(g) cmt 7: requires the defendant to be informed through his lawyer -Burt's confession, combined with the experience of his blood at the scene of the crime, would give Daniel knowledge of 'new, credible and material evidence creating a reasonable likelihood' that Ray did not murder Tammy. This triggers the duties spelled out in Rule 3.8(g)

The police arrested a middle school teacher for repeatedly sexually abusing numerous students. Some of the victims told the police that they had reported the abuse to the school principal, but the principal failed to take action. After the teacher was arrested, some of the victims and their families filed suit against the principal and the school board. The school board hired the firm of S&K to 'investigate the response of the school administration" to get a full report so that it could decide whether to settle or litigate the claims. Meanwhile, the school board also hired a respected trial attorney, Lindsay, at another firm to represent the board in the lawsuit. S&K interviewed all the officers of the school board. Their notes and report were turned over to the school board and Lindsay. The plaintiffs in the lawsuit requested, during discovery, the notes made by S&K during their interviews. Lindsay must:

-refuse to turn over the notes to the plaintiff b/c they are protected by attorney client privilege -Upjohn case--the lawyer collecting information for the client so the client can mount a legal defense -it is irrelevant that a different law firm is representing the client in the lawsuit

Attorney Alton recently filed a medical malpractice action on behalf of Juan against Metropolitan Hospital, where Juan was sexually assaulted by a staff member while he was being treated for a psychotic episode in the psychiatric ward. Juan was traumatized by his experience at the hospital. In a telephone call before their first meeting, Alton learned that Juan suffers from post-traumatic stress disorder and needs a social worker to be present in all stressful situations. Alton therefore asked Juan to bring his social worker, Pierre, to all of their metings. Juan pays Pierre to assist him, and he brought Pierre to all of his meetings will Alton. The lawyers for Metropolitan Hospital have learned about Pierres presence during the meetings between Juan and Alton. The hospital is seeking discovery of the conversations that took place during those meetings. Which one of the following statements is correct?

-the attorney client privilege would have protected the conversation b/w Alton and Juan from discovery despite Pierre's presence even if Juan had brought him to the meeting w/o any suggestion from Alton -Pierre's presence in the meeting was necessary for Juan to be able to communicate comfortably with Alton -Rest. 70: the presence of a third party can cause a conversation b/w Lawery and the client not to be found privileged, but an exception exists when the third party is necessary to the conversation.

Packard manages an apartment building where Manuel is a tenant. Tom handles legal matters for Packard. Manuel hasn't paid rent and Packard wants to evict him. Manuel called Packard complain about the building's services but never asserted he was declining to pay rent because of this. Packard's policy directs Tom to make one effort to negotiate for full payment within 30 days of commencing eviction proceedings. Tom's friend Ivan is Manuel's boss at Ecosystem. Ivan told Tom that Manuel hired a lawyer named Serena to sue Ecosystem for race discrimination. Tom is planning to identify himself Packard's lawyer, but is not planning to ask Manuel whether counsel represents him, or to advise Manuel that he has a right not to speak or to be represented with the threatened eviction. After identifying himself, Tom will ask why he hasn't paid rent and then negotiate for payment. May Tom have this conversation with Manuel?

-yes b/c Tom does not know that Serena is representing Manuel in his dealings with Packard -Rule 4.2 prohibits communications about the subject of the representation with a person the lawyer knows to be represented 'in the matter' unless the lawyer has the consent of that persons lawyer -Rule 4.2 cmt 8: a lawyer may not evade the requirement of obtaining the consent of counsel by closing his eyes to the obvious but the rule does not require the lawyer to ask if Manuel has a lawyer in the matter -If Tom does not know that Manuel has a lawyer in connection with his non-payment of rent, he may communicate with Manuel

Nathan is a partner in the law firm of Pace and Gillespie, which represents Bostwick Corp., which is not publicly traded. The corporation manufactures boots. Nathan is a friend of David Bostwick, the chairman of his client's board of directors, who owns 12 percent of the stock of the company. Nathan himself owns 3 percent of the stock, which he acquired long before Pace and Gillespie began representing Bostwick. Whenever there is a vacancy on the board, the board selects a replacement director. No directors are elected by the shareholders. On David's recommendation, the board takes a formal vote and offers Nathan the opportunity to join the board. Nathan would like to accept, and neither he nor anyone on the board knows of any actual or apparent conflict of interest. Nathan is not willing to divest himself of his stock in the company. May Nathan accept the offer to join the board?

-yes, b./c there is no law or ethical rule barring his joining the board -Rule 1.7 cmt 35: in the absence of an actual or apparent conflict, a lawyer may serve on the board of a corporation that the lawyer represent and the lawyer may may also own stock in the corporation -the lawyer should be aware that communications with corporate employees may not be covered by attorney-client privilege and the lawyer should recuse himself or resign from the board if a conflict arises

In 2009, Anita represented Liam, an immigrant from Australia who was charged with selling cocaine. Under state law, if convicted, Liam could have been sentenced to 5 years in prison. The DA offered a deal providing that if Liam pled guilty, he would be sentenced to a term of 14 months. W/o researching potential immigration consequences, Anita encouraged him to accept the offer, and he did. At the end of his sentence, the state turned Liam over to federal immigration authorities, who deported him back to Australia based on his criminal conviction. Liam cannot show that he would not have been convicted had it gone to trial. May Anita be subject to discipline?

-yes, b/c Anita failed to investigate the potential immigration consequences of pleading guilty and failed to advise her client about that -Rule 1.1 -Anita's failure to investigate and advise her client about the immigration consequences of a conviction for her immigrant client is incompetent and she could be subject to discipline

Bobby, a former employee of Talmart, Inc., has retained Ashley to bring an employment discrimination action against Talmart, claiming that his firing was unlawful discrimination based on religion. Ashley is investigating the matter. Ashley wants to interview Chi, a former employee who left the company 2years ago. Chi was Talmart's HR VP; she was responsible for hiring, promotion, and discharge of employees. Bobby told Ashley he was fired after he complained about the policy prohibiting employees from wearing turbans. Bobby's Sikh faith requires him to wear a turban. Ashley wants to contact Chi to ask about Talmart's turban policy, without first informing Ned, the lawyer who Ashley knows represents Talmart in all employment-related litigation. Assuming that Ashley would identify herself to Chi as Bobby's lawyer, may she interview Chi without seeking permission from Talmart's lawyer?

-yes, b/c Chi no longer works for Talmart -Rule 4.2 cmt 7: a lawyer may not contact an employee of a corporation who 'has authority to obligate the organization with respect to the matter or whose act or omission in connection with the matter may be imputed to the organization' and 'consent of the org's lawyer is not required for communications with a former constituent' -THUS a lawyer may contact any former employee WITHOUT Talmart's consent

Attorney Pavel is a criminal defense lawyer representing Denny, who is charged with larceny. Denny, a pro bono client, is alleged to have stolen a piece of jewelry from a jewelry box during a house party. In an unrelated matter, Pavel is defending Betty, for a fee, against a criminal charge of arson. Both cases are assigned to be heard by a single judge. During Denny's trial, the prosecutor calls Betty to the stand. She testifies that she was at the party in question and saw Denny go in and out of the room where the alleged theft occurred. She further testifies that she never went into that room. Earlier, Denny had told Pavel that he saw Betty go into the room, and he suggested that she may be the thief. Pavel intends to cross-examine Betty on this point. Is there a conflict of interest?

-yes, b/c Denny's interests are directly adverse to Bettys -Rule 1.7 cmt 6: a directly adverse conflict may arise when a lawyer is required to cross-examine a client who appears as a witness in a lawsuit involving another client, as when the testimony will be damaging to the client who is represented in the lawsuit -Pavel must cross-examine Betty in order to represent Danny competently and diligently. Betty's testimony may exculpate Danny and inculpate Betty. After cross, Danny might be acquitted on the charge and Betty indicted. Even if she is not charged with the jewelry theft, the cross could negatively impact her credibility and her potential liability on the arson charge

Ellen wants to hire Donna, an attorney, to handle her divorce case. Ellen works as a saleswoman in a department store, earning $26,000 a year. Donna proposes to charge a fee of $300 per hour, which is not an unusual hourly rate for divorce work in Ellen's community, although some lawyers charge less. Donna discloses her fee and all expenses for which Ellen will be responsible in writing, and Ellen signs the writing. May Donna charge this fee?

-yes, b/c Donna informed Ellen before starting to work that her fee will be $300/hour and listed the expenses for which Donna will be responsible -Rule 1.5(j): requires the basis or rate of the fee and expenses for which the client will be responsible to be disclosed

Ella, a public defender, represented a college student named Kenneth. Kenneth was driving his brother car and was stopped. The officer spotted what looked like cocaine on the front seat and arrested Kenneth. When the substance was tested, it turned out to be cocaine. Against Ella's advice, Kenneth told Ella that he planned to plead guilty. Ella writes a blog about her experiences as a public defender. After Kenneth was released pursuant to a negotiated deal for deferred prosecution, she wrote on her blog: * "#126409 (the client's jail id #): This stupid kid is taking the rap for his drug-dealing dirtbag of an older brother because 'hes no snitch: I managed to talk the prosecutor into treatment and deferred prosecution. My client is in college. Just goes to show that higher education does not imply that you have any sense." Jail ID #'s are not available to the general public. Is Ella subject to discipline?

-yes, b/c Ella revealed confidential information -the jail identification number is not available to the general public, but jail officials and perhaps police and prosecutors could match the number with the clients identity. Even if she had omitted the number, these officials or others in the criminal justice system might be able to piece together which of her clients Ella was referring to, and would be able to learn a great deal of private information about Kenneth. Both the jail ID number and her other comments make her revelation improper

Josh, a 3L, is planning to apply for admission to the bar. He just looked at the NCBE questionnaire. It asks: Have you ever been dropped, suspended, warned, placed on scholastic or disciplinary probation expelled, requested to resign, or allowed to resign in lieu of discipline from any college or University, or otherwise subjected to discipline by any institution or requested or advised by any institution to discontinue your studies there? 4 years ago, a RA found alcohol in Josh's dorm room. At a meeting, Josh agreed to be put on probation for one semester. If he made it through the semester without any infractions, the incident would be expunged from his record. He completed the semester without incident, and the matter was expunged. His law school asked a similar question on its application, and he answered "no". Should Josh reveal the above-described incident in his character and fitness questionnaire?

-yes, b/c Josh was placed on disciplinary probation by his college, so the question calls for disclosure of the incident and includes no exemption for 'expunged' offenses -Josh must reveal the incident and provide any requested details. Minor transgressions in the past are unlikely to derail a bar application but answering a question falsely on a bar application would violate Rule 8.1(a) and could lead to denial of admission or to discipline if the false statement were ever discovered

Kieran, 5 years out of law school, accepted a job with S&P, LLP. A couple months later, Miranda was asked to take over representation of Kasho in an ongoing lawsuit against Roxbury. Kieran came to S&P, LLP from another firm, P&A LLP, where he had worked for 3 years. P&A LLP has been representing Roxbury in the Kasho litigation for the last 2 years. Kieran did not work on the case but was dating Lance, the senior paralegal at the firm who oversaw all the staff work on the Roxbury case and talked at length about the case with Kieran. After consultation her firm's ethics counsel, Miranda decides to represent of Kasho. She timely screens Kieran from the lawyers working on the matter, sees to it that he receives no part of the fee from the Kasho matter, and provides appropriate written notices to Roxbury about the screening procedures. Was S&P, LLP permitted to undertake representation of Kasho in this manner?

-yes, b/c Kieran was properly screened form the Kasho representation -Rule 1.9(b) -Miranda may go forward only if Kieran does not work on the matter and receives no part of the fee from it, and the firm implements all of the screening procedures required by Rule 1.10(a)(2) -Kiernan himself did not represent Roxbury, it was a client of his former firm. This rule would bar Keiran from working on this matter unless Roxbury gave informed consent b/c (1) the matters are the same or substantially related, (2) the interests of the parties are materially adverse, and (3) Kieran knows confidential information about the former matter that is material to the new matter

Tax attorney Tammy represents client Lilian, who plans to buy several tickets to upcoming concerts using her student ID and sell them to non-students for a significant profit. She seeks Tammy's advice as to whether she must pay income tax on the money she expects to make. She does not know that reselling student tickets for profit is illegal in her state, and when Tammy informs her that it is, she does not go forward with the plan. Is Lilians conversation with Tammy privileged?

-yes, b/c Lillian did not go through with her plan -the communication is privileged and the crime fraud exception does not apply b/c Lillian did not go through with the crime

Milan graduated from law school, took the bar exam in state A, passed the exam, and was admitted to the bar in state A. He maintains active membership even though he is neither practicing law, nor living in state A. Milan bought a small computer consulting firm in neighboring state B and ran it successfully for a few years, earning a good living. Then Milan decided to sell the business. He found a buyer who purchased the business. In the course of their discussions about the business, Milan represented that the business had been twice as profitable as it actually was. After a few months of operating the business, the buyer sued Milan for damages for fraud and to rescind the contract. That lawsuit is pending. Is Milan subject to discipline in state A?

-yes, b/c Milan lied to the buyer about the value of the business, even though his dishonesty took place in state B -Rule 8.4: prohibits dishonesty, fraud, deceit, or misrepresentation in our outside of the practice of law -Rule 8.5(a): a lawyer admitted to practice in this jurisdiction is subject to the disciplinary authority of this jurisdiction, regardless of where the lawyers conduct occurs

Sol practices tax law, but his expertise is in giving tax advice rather litigation. Recently, Sol provided personal tax advice to Bill. Bill then asked Sol to look at a tax matter. Bill wants Sol to litigate the matter in the tax court. Sol mentioned to his friend Al, a litigator, that one of his clients wants him to take on a litigation matter. Al responded, "Just tell your client that you will do it with co-counsel. I will charge Bill 33% of the recovery, and I'll give you half of what he pays us. We'll both enter appearances and sign the papers. It will be as if we were in a law partnership together!" Sol and Al agree to accept any liability for mistakes. The overall fee they intend to charge is reasonable. Bill gives his informed consent to Al's association with Sol, to the proposed fee, to how Sol and Al will split it, and he signs a writing to that effect. May Sol and Al enter into this split-fee arrangement?

-yes, b/c the arrangement complies with all the relevant rules -Rule 1.5(e) -this arrangement complies with 1.5(e) on division of fees b/w lawyers not in the same firm, but the lawyers must also comply with rule 1.5(c) b/c this is a contingent fee agreement -1.5 cmt. 8: contingent fee agreements must be in writing signed by the client and must otherwise comply with paragraph c of the rule

Paula, a lawyer, drafted a will for Viet. The will stated that in the event of his death, his property should go to "my children." Viet and his wife Jenny had two sons. After his death, Edith, a woman with whom he had had an affair, proved through DNA evidence that he also had fathered her son Michael. She claimed that her son was entitled to one third of the estate. During probate of the will, Paula plans to testify that Viet had told her that what he meant was that his property should go to his children Gabriel and Veronique, and any children that he and Jenny later produced. Since Jenny is next of kin, Paula has obtained her informed consent to this plan. Paula would assert that she was waiving the privilege on behalf of her deceased client. Edith's lawyer objects on the ground that Paula's testimony would violate the attorney-client privilege. Should the probate judge allow Paula to testify about Viet's intent?

-yes, b/c Paula's testimony would fall under an exception to that privilege -Swidler & Berlin v. US: there is a common law 'testamentary' exception to the privilege for disputes among heirs. The court explained that the interest in settling estates outweighs the privileges that would otherwise be accorded to deceased persons who communicated with their attorneys and that revelation in estates cases furthers the effectuation of a deceased client's intent

Attorney Seamus represents Rona in her purchase of an apartment in a major metropolitan city. The apartment is priced well below market value, and it seems to both Seamus and Rona that it is a very good deal. Seamus learns that the apartment is priced so low because one of the bedrooms was constructed below ground level in violation of the city's housing code. When Seamus relays this information to Rona, she gets cold feet and decides not to sign the contract. She finds another apartment and closes a purchase agreement. As it happens, Seamus is also looking to buy an apartment. The apartment that Rona declined to buy would work well for him and his family. He is also willing to buy the apartment knowing that it has a bedroom that does not meet the code requirements. He talked with Rona about his interest in the apartment. She says it would not concern her at all if he bought it. May Seamus purchase the apartment?

-yes, b/c Rona no longer wants to buy the apartment -Rule 1.8(b): prohibits the use of confidential information to the disadvantage of the client -Here, Rona has decided not to buy the apartment; there is no disadvantage to the client -Rule 1.8 cmt 5 explains that the rule does not prohibit uses of confidential information that do not disadvantage the client

Lucille, a criminal defense lawyer, receives a call from Svlvia, who says her sister, Basia, who immigrated from Poland, has been arrested, and that the bail hearing will be held in less than 2 hours. She asks Lucille to represent Basia at the bail hearing and in her case. Lucille hires Magda, an interpreter, and rushes to the courthouse, where she is able to meet with and represent Basia. The next day, Lucille meets with Basia, and informs her that her rate is $300/hr. She states that that rate will be used to calculate work Lucille did and for future work. Basia agrees. There is no other discussion of how much Basia will have to pay. At Basia's hearing five months later, the arresting officer fails to appear, and the charges are dropped. The next month, Lucille sends Basia a bill for $1,800, including 5 hrsof her work, plus $300 for Magdas services. Is Lucille subject to discipline?

-yes, b/c she did not communicate the expenses for which Basia will be responsible w/in a reasonable time after commencing the representation -Rule 1.5(b): a lawyer must communicate to the client the basis or rate of the fee and expenses for which the client will be responsible w/in a reasonable time after commencing representation. -Lucille billed Basia for Magda's services, even though she had not informed her that she would do so

Donald was suspected of robbery. Shortly after his arrest, he demanded to see his lawyer, Craig. The police didn't question Donald, and Craig arranged for his release on bail after Donald was read the charges and given a court date. Craig filed papers informing the court and the prosecutor that he was representing Donald. Brenda, a prosecutor, was assigned to Donald's case. A state statute authorizes prosecutors to investigate criminal cases by questioning witnesses or authorizing police to do so up to the point charges are filed. Brenda visited Donald's home a week later, identified herself, and asked if she might ask some questions. Donald said he would answer her questions. Brenda didn't make false or misleading statements. On the basis of his answers, Brenda was able to contact and interview other witnesses. She didn't offer into evidence any info that she received from Donald. Is Brenda subject to discipline?

-yes, b/c she did not first obtain Craig's consent to talk to Donald Rule 4.2: requires consent of opposing counsel before a lawyer, including a prosecutor, to talk to an opposing party known to be represented by counsel in the matter

Herman was mugged, and his wallet, which contained $200 in $20 bills, was stolen. Herman spotted a police officer and reported the crime. He described the assailant male with dark hair, medium height, and a dark shirt and white sneakers and said the assailant had a knife. The officer alerted other officers. Juan was arrested 3 blocks away. He has dark hair and medium height, was wearing a dark blue shirt and white sneakers. When searched, he was found to have $454 in cash, including 8 $20 bills. He had no knife. When arrested, Juan acted suspiciously, and refused to answer any questions, and didn't confess. Herman's wallet was never found, and no knife was found. The case was assigned to prosecutor Marcella. She believes that Juan is guilty, though she doubts a jury would find that. She is uncertain whether the charge is even supported by probable cause. Is it proper for Marcella to charge Juan with robbery?

-yes, b/c she does not know that the charge is not supported by probable cause -Rule 3.8(a): bars a prosecutor from moving forward with a charge that the prosecutor 'knows' is not supported by probable cause -Rule 1.0(f):Knows denotes actual knowledge of the fact in question -Marcella is uncertain about whether the probable cause standard is met, so she does not 'know' the charge would be unsupported by probable cause

Attorney Maria is a divorce lawyer. Several months ago, Maria represented Jillian in her divorce from her husband, Jack. At that time, Maria charged her an hourly fee of $400 per hour. The judge granted the divorce and ordered Jack to pay her $1,000 per month child support. Recently, Jillian came to Maria seeking help, because Jack has not been paying the child support for the past 6 months. She would like Maria to help her get the $6,000 that Jack owes her. Maria wishes to charge Jillian a contingent fee of 30 percent of the recovery. She discloses the fee and expense terms and receives Jillian's informed consent in writing. May Maria charge the contingent fee?

-yes, b/c she obtained Jillian's consent in writing -Rule 1.5(c): a lawyer may enter into a contingent fee arrangement with a client as long as the client gives consent in a signed writing

Jennifer is a partner , and she has been practicing law for 8 years. She has handled primarily trusts and estates cases. She has taken a domestic relations case representing Eunice, who is seeking a divorce from her husband, Rudy. Jennifer discovered evidence that indicated Rudy was sexually abusing Eunice's daughter. Eunice does not want Jennifer to report this to authorities, because she doesn't believe that it is true. Jennifer is not sure whether she has a duty to report the apparent child abuse to the state authorities pursuant to a state reporting statute. Nobody else in Jennifer's firm has any experience with family law. A friend of Jennifer, Isai, is an experienced domestic relations lawyer in a different law firm but is not her firm's ethics counsel. Jennifer has not talked with Eunice about consulting another lawyer. May Jennifer consult Isai about whether she has a duty to report the sexual abuse?

-yes, b/c the purpose of the disclosure is to obtain advice about whether Jennifer must report the child abuse -Rule 1.6(b)(4): a lawyer may reveal confidential information outside of the law firm for the purpose of securing advice about her ethical obligations -Jennifer could consult Isai about whether she has a duty to report under the state child abuse reporting statute and rule 1.6(b)(6) or whether she is obliged not to report by 1.6(a). Although the rule permits Jennifer to reveal confidences to obtain advice about her ethical duties, she would be well-advised not to reveal the client's identity or identifying information to consult Isai. Instead, she might frame the factual basis of the consultation as hypothetical

Caoline represents Frostco, Inc.. She receives a visit from Mark, a Frostco employee. Mark says that the sales contracts Caroline prepared for Frostco included fraudulent misrepresentation about the durability of fridges that Frostco was selling to distributors. Mark provides Caroline with secret internal memos in which company officers acknowledged the substandard compressors. Caroline confronts the general counsel of Frostco, who directs her to back off and continue prepping the contracts. He declines to address the product defects or to scale back the warranty claims in new contracts. He says, "if you don't want our business, we will find another lawyer" Caroline decides to withdraw from the representation. May Caroline tell the distributors that her client misrepresented the durability of the fridges?

-yes, b/c the rules permit disclosure in these circumstances so that the distributors can try to recoup their losses -Rule 1.6(b)(3): a lawyer may reveal confidences to the extend the lawyer reasonably believes necessary to prevent, mitigate, or rectify substantial injury to the financial interests of another that is reasonably certain to result from the clients commission of fraud in which the lawyers services were used

Tandy is a practicing lawyer and adjunct law professor at Frostburg Law School, which is part of Frostburg University. Two Caucasian high school seniors have asked her to represent them in their lawsuit against Frostburg University. They allege that Frostburg University did not admit them to the undergraduate program because of race discrimination. Specifically, they allege that students of color with similar test scores and grades were admitted to Frostburg University, while they were not. Tandy reasonably believes that she can competently and diligently represent the students in their lawsuit. She obtains informed consent, confirmed in writing, from both the students and the university. May Tandy represent the two students in their lawsuit?

-yes, b/c the students gave informed consent, confirmed in writing -Rule 1.7(a)(2): material limitation -the students consent is required b/c there is a significant risk that Tandy's representation of them will be materially limited by her obligations to her employer

Last year, attorney Mira represented Sally, the owner of a small pizza restaurant, in a suit against Rome's Own, Inc., a supplier of shredded cheese, based on breach of contract. The parties settled the claim for $15,000. Mira and Sally remained friendly after the matter was settled, and have gone to the movies and to dinner a few times in the last year. Earlier today, Sally's next-door neighbor, Jose, came into Mira's office for a consultation and asked Mira to represent him in a civil suit against Sally. Jose alleges that Sally's pet macaw bit him, causing an infection and permanent scarring. Jose wants to get a court order against Sally requiring that the bird to be euthanized pursuant to a state law. Sally already paid for his medical care. Jose does not want further damages. Must Mira obtain Jose's informed consent before she can agree to represent Jose?

-yes, b/c there is a significant risk that Mira's representation of Jose will be materially limited by her loyalty to Sally -Rule 1.7(a)(2): a lawyer must obtain the consent of a prospective client if 'there is a significant risk that the representation of that client will be materially limited by the lawyer's responsibilities to a former client -Mira and Sally had a professional relationship and have had continuing social contact, so it is possible that Mira would be less vigorous in her advocacy on behalf of Jose b/c of her relationship with Sally

Larry, the father of Linda, Abby, and Carl died. His property was to be divided equally among his children. Most of his property is in cash and stocks, but Larry also left his children a lakeside cabin. The three adult children decide that Carl should take the cabin and should pay his sisters 1/3 of its value each out of his share of the cash and stocks. The appraised value of the cabin is $225,000, but Carl urges that the cabin should be valued at $150,000 b/c he believes he will spend considerable cost on repairs. Linda and Abby are willing to do so. Carl proposes that they hire Morton. Morton reasonably believes that he can competently represent all three siblings. He explains the potential problems, advantages, and risks and obtains their consent to the joint representation. He did not provide any legal advice to them. Morton then prepares a document valuing the cabin at $150,000 for purposes of the transfer, jus

-yes, b/c there is no apparent conflict b/w the interests of the three siblings and he obtained their informed consent -Rule 1.7 CMT 18: When the representation of multiple clients on a single matter is undertaken, the information must include the implications of the common representation, including possible effects on loyalty, confidentiality, and the attorney-client privilege and the advantages and risks involved. -Morton's prior relationship with Carl and the somewhat divergent interest of the siblings may create an actual or potential conflict under Rule 1.7(a)(2) -BUT if all thee give informed consent, Morton may represent all three b/c the siblings interest are not fundamentally antagonistic to each other but are generally aligned in interest even tho there is some divergence in interest among them

Carlos, a sole practitioner, represents Sharky Products, Inc., a property owner that disputes an adjoining owner's right of access to an alleyway between their buildings. Carlos sent a letter to the other property owner, Berry's Beauty Supply Co, to explain the problem. He received a reply from Berry's lawyer, Eleanor, requesting that Carlos withdraw from representation of Sharky. Eleanor claimed that Carlos was prohibited from handling this matter because five years ago, Carlos represented Matthew, the president and sole owner of Berry's, in a claim against the manufacturer of a lawn care product that he used at his home. The matter took three hours of Carlos' time and Matthew was satisfied with the resulting settlement. Carlos has had no contact with Matthew since then. May Carlos continue to represent Sharky Products, Inc.?

-yes, b/c there is no substantial relationship b/w the two matters -Rule 1.9 cmt 3 -the prior representation was of Matthew as an individual in a matter that would not normally have involved 'a substantial risk that confidential factual information as would normally have been obtained in the prior representation would materially advance the clients position in a subsequent matter'. This is evident b/c the prior representation was not of Sharky but of one of its officers and the representation did not involve financial information about the corporation

Carla, a graduating law student who is in the US on a student visa, is having lunch with Alice, a lawyer who graduated from the same law school the previous year and was just admitted to the bar. Carla tells Alice that her student visa is about to expire, but that she has just been offered a job at a law firm. She asks whether the law firm could sponsor her for a visa. Alice responds, "I don't believe so?" Carla, who reasonably believes that Alice was providing correct legal advice, turns down the law firm job and returns home. A few months later, she learns that Alice's advice was incorrect. Alice's advice was negligent and was the cause of Carlas return to her home country and failure to get the job. Alice never signed a retainer, paid a fee, or otherwise agreed to be Carlas lawyer. Carla sues Alice for negligence. May a court grant judgment against Alice for malpractice?

-yes, because Carla reasonably believed that Alice was providing correct legal advice -Under Togstad v. Vesely, Otto, Miller & Keefe, an attorney-client relationship is formed when an individual seeks and receives legal advice, and a reasonable person would rely on that advice. If an attorney-client relationship existed, the plaintiff need only show that the attorney acted negligently, that such acts were the cause of plaintiff's damages, and that but for the negligent advice, she would not have suffered damage

Seymour owns and manages a small dairy farm. Recently, Seymour discovered that his fertilizer supplier had been mislabeling its fertilizers as organic when, in fact, they were not. Seymour has had to recall many of his products that were labeled as organic, at great expense, and his reputation has suffered as a result. He wants to sue the supplier for damages. He meets with attorney Gemma to see whether she will represent him. Gemmas usual fee is $200 per hour. Knowing that Gemma is one of the best lawyers in town, Seymour offers to pay her $250 per hour for her services. Gemma has the time and requisite knowledge and experience in the field to represent Seymour. However, as a vegan, she would find it morally repugnant to represent a dairy farmer. May Gemma refuse to represent Seymour?

-yes, because a lawyer is not obliged to accept a client whose cause the lawyer regards as repugnant -Rule 6.2 cmt 1 -a lawyer ordinarily is not obliged to accept a client whose character or cause the lawyer regards as repugnant

Lawyer Danielle. Ted, the son of Her brother Matthew, is very ill. Ted must get a medication that costs $1,400. Matthew has no other possible source of funds unless Danielle gives him a short-term loan. Danielle is flat broke, but there is $120,000 in Danielle's trust account. This amount is from a settlement Danielle will distribute to her client Van; she represented him pro bono. Van is in prison. Danielle has agreed to hold Van's funds and to make mortgage payments on his house. Danielle lends Matthew $1,400 from this account for Ted's medication. She is unable to ask for Van's approval. Danielle sends him a letter explaining. A week later, Matthew receives the proceeds from the sale of his house and repays Danielle, who replenishes the account. Van later writes Danielle that he was okay w/ this. There is no resulting delay in the distribution or in the making mortgage payments. Is Danielle subject to discipline?

-yes, even tho she promptly informed Van that she borrowed the money and he ratified her prior action -a lawyer may not use money from a client trust account for her own purposes, even for humanitarian reasons

Sam, a lawyer, represents Alan, who is in jail facing assault charges from a bar fight with Don. Sam goes to witness Toby's house. *questioning* S: I am a lawyer for Alan, and I am investigating the fight two weeks ago. I understand that you were there. T: That's right. S: Is a lawyer representing you in this matter? T: No, I was not involved. S: Could you tell me what happened? T: T two guys had been drinking and started arguing. S: Where were they and where were you? T: They were at a table. I was on the barstool nearest to them. S: Could you hear what they were saying? T: I can't remember. But I remember the fistfight. I saw that pretty clearly. S: Have you talked to anyone about this? T: No, I thought about going down to your station, but I didn't want to get involved with police. Sam wants to ask who threw the first punch. Would he be subject to discipline if that is the next thing he does?

-yes, v/c he must first make a reasonable effort to correct Toby's confusion about his role -Toby is evidently under the misimpression that Sam is a police investigator, as evidenced by the phrase "your station." Sam must clarify his role to correct Toby's misimpression


Kaugnay na mga set ng pag-aaral

Ch. 12: The Reach of Imperialism

View Set

Molecular and Cellular Biology - Exam 1 Test Bank 3

View Set

Applied Management Module 3 Mastery Quiz

View Set

AP Statistics Semester 1 Quiz/Checkpoint Questions GOD SEND

View Set

Psychology Quizzes Multiple Choice

View Set

Stigma - social aspects, understanding illness from patient's perspective

View Set